Is 1.875 and rational number

Answers

Answer 1

Answer: 1.875 is a rational number because it can be represented as a ratio or a fraction.


Related Questions

HURRY!!! ILL MARK U BRAINLIEST!! After completing the fraction division StartFraction 7 Over 3 EndFraction divided by StartFraction 2 Over 9 EndFraction, Bai used the multiplication below to check his work. StartFraction 21 Over 2 EndFraction times StartFraction 2 Over 9 EndFraction = StartFraction 42 Over 18 EndFraction = StartFraction 7 Over 3 EndFraction Which is the most accurate description of Bai’s work? Bai found the correct quotient and checked his work using multiplication correctly. Bai found the correct quotient but checked his work using multiplication incorrectly. Bai found an incorrect quotient but checked his work using multiplication correctly. Bai found an incorrect quotient and checked his work using multiplication incorrectly.

Answers

Answer:

The answer is a.

Bai found the correct quotient and checked his work using multiplication correctly.

Step-by-step explanation:

I know this cuz I did the test on edge :))

Anyways plz give me the brainliest!!

Answer:

ITS A

Step-by-step explanation:

i did it on edge 2020

pls give Brainliest.

[GIVING BRAINIEST] What is 18/6

Answers

Answer:

3

Step-by-step explanation:

18/6 = 3

90 + 12 = 6(15 + 12)


Answers

The answer for your question is 102

The parking lot at a grocery store has 90 cars in it. 90% of the cars are blue. How many cars are blue?
a 10 cars
b 81 cars
c 810 cars
d 100 cars

Answers

Answer:

81 blue cars

Step-by-step explanation:

Total Cars = 90

How much % are blue = 90

=> 90 x 90/100 = How many cars are blue

=> 8100/100 = the number of cars that were blue

=> 81 = cars that were blue.

There 81 blue cars in the parking lot.

what would be the answer to g+3=17

Answers

Answer:

g = 14

Step-by-step explanation:

g + 3 = 17

minus 3 from both sides

g = 14

Peter plays on the varsity basketball team. So far this season he has scored a total of 52 points. He scores an average of 13 points per game. How many more games must Peter play in order to score 117 points?​

Answers

Answer:

5 more games.

Step-by-step explanation:

117 - 52 = 65.

65 / 13 = 5.

Which model represents a fraction less than Three-fifths? A circle divided into 5 equal parts. 4 parts are shaded. A circle divided into 5 equal parts. 2 parts are shaded. A circle divided into 5 equal parts. 6 parts are shaded. A circle divided into 10 equal parts. 8 parts are shaded.

Answers

Answer:

The correct answer is:

A circle divided into 5 equal parts. 2 parts are shaded.

Step-by-step explanation:

To determine the fraction less than three-fifths, let us convert the word problems into figures and compare the values:

[tex]Three-fifths = \frac{3}{5} = 0.6[/tex]

a) A circle divided into 5 equal parts. 4 parts are shaded (four-fifths)

[tex]Four-fifths = \frac{4}{5} = 0.8\ (more\ than\ three-fifths)[/tex]

b) A circle divided into 5 equal parts. 2 parts are shaded. (two-fifths)

[tex]Two-fifths = \frac{2}{5} = 0.4\ (less\ than\ three-fifhs)[/tex]

c) A circle divided into 5 equal parts. 6 parts are shaded = [tex]\frac{6}{5} = 1.2[/tex] (more than three-fifths)

d) A circle divided into 10 equal parts. 8 parts are shaded [tex]= \frac{8}{10} = 0.8\ (more\ than\ three-fifths)[/tex]

Therefore, the fraction less than three-fifths = A circle divided into 5 equal parts. 2 parts are shaded. (two-fifths)

A circle divided into 5 equal parts. 2 parts so the answer is B

explanation:

Hope this works<3

Two angles from a liner pair. The measure of one angle is 4 times the measure of the other angle. Find the measure of each angle

Answers

Answer:

x = 36 degrees, the smaller angle.

4(36) = 144 degrees, The larger angle.

36 + 144 = 180 degrees.

Cleomenius.

Answer:

x = 36 degrees, the smaller angle.4(36) = 144 degrees, The larger angle.36 + 144 = 180 degrees.

Step-by-step explanation:

(1)adjacent- two non-overlapping angles that share a common vertex and a common side

(2)the non-common sides are opposite rays.

(3)adds up to 180

-2, -1, -1, 0, -1, 1, -2, … Find the next 3 numbers in the sequence above. Describe the pattern.

Answers

Answer:  -2, -1, -1, 0, -1, 1, -2, -1, -1, 0

Step-by-step explanation:

When you are looking at the problem

" -2, -1, -1, 0, -1, 1, -2, … Find the next 3 numbers in the sequence above. Describe the pattern."

you want to observe the pattern, now For example

1, 2, -1, -4 .. and so on

so there is a pattern here if looked at hard enough it goes 1 - 2 = -1 - -3 = -4

when you find the pattern it gets really easy.

Hope this helped!

Describe the steps to copy angle CAB∠ C A B.

Answers

Answer:

Step-by-step explanation:

Construction: To copy angle CAB.

i. On angle CAB, with center A and any radius; draw an arc to intersect CA at E and AB at F.

ii. Place a divider at EF to copy the angle.

iii. Draw a separate straight line MN of approximate length to AB.

iv. Mark point P on MN. Place the divider with radius EF at P to mark Q.

v. Draw a line LM from M through Q.

vi. Therefore, angle CAB would be equal to angle LMN.

The difference of the squares of two positive consecutive even integers is 12. Find the integers. Use the fact that, if x represents an even integer, then x + 2 represents the next consecutive even integer

Answers

Answer:

2 and 4.

Step-by-step explanation:

(x+2)^2 - x^2 = 12

x^2 + 4x + 4 - x^2 = 12

4x + 4 = 12

4x = 8

x = 2.

Hence the integers are 2 and 4.

Jackson drops a pumpkin off a balcony from
a height of 64 feet. Analyze the time in seconds it takes
for the pumpkin to smash on the ground.

Answers

Answer:

t = 1.15 seconds.

Step-by-step explanation:

The pumpkin is dropped from the height of = 64 feet.

Now we have to find the time taken to reach the pumpkin on ground. Use the below formula to find the time taken.

H = ½ gt^2

H = height

g = 9.8 (always contant )

t = time taken.

Now insert all the values in the formula.

H = ½ gt^2

64 = ½ ×9.8× t^2

(64×2) / 9.8 = t^2

t = 1.15 seconds.

Between which two integers does square root of /51 lie

Answers

Answer:

7 and 8

Step-by-step explanation:

Consider the perfect squares either side of 51

49 < 51 < 64 , thus

[tex]\sqrt{49}[/tex] < [tex]\sqrt{51}[/tex] < [tex]\sqrt{64}[/tex] , that is

7 < [tex]\sqrt{51}[/tex] < 8

Answer:

7 and 8.

Step-by-step explanation:

7^2 = 49 and 8^2 = 64.

Find and describe the error in the followig work -3(2x+3)=21 -6x+9=21 -6x=12 x=-2

Answers

Answer:

-3(2x+3)=21 does not become -6x+9=21 because wrong foiling.

Step-by-step explanation:

When you distribute -3 with 2x+3 you multiply both values by -3.

-3*2x = -6x

-3*3 = -9 <- This is where the error is. It's -9, not +9.

Janice published a novel. Last month the book sold 1326 copies, earning Janice a total of $1579.76. This month the book sold 1445 copies. How much should Janice expect to receive in royalties this month?

Answers

$1,721.53. That’s your answer.

Determine whether the equation below has a one solutions, no solutions, or an infinite number of solutions. Afterwards, determine two values of x x that support your conclusion. x − 5 = x−5= 5 − x 5−x

Answers

Answer:

It has 1 solution

[tex]x=5[/tex]

Step-by-step explanation:

Given

[tex]x - 5 = 5 - x[/tex]

Required

Determine the number of solution

[tex]x - 5 = 5 - x[/tex]

Collect Like Terms

[tex]x + x = 5 + 5[/tex]

[tex]2x = 10[/tex]

Divide both sides by 2

[tex]\frac{2x}{2} = \frac{10}{2}[/tex]

[tex]x = \frac{10}{2}[/tex]

[tex]x = 5[/tex]

Since the value of x is 5, then the equation has 1 solution

[tex]x = 5[/tex] is its only value

Please help!!! I'll mark you brainiest and give you as many Points as possible!!!!!

Answers

Answer:

Option C

Step-by-step explanation:

[tex]f(x) = 40( \frac{1}{4} )^{x} [/tex]

Answer: its a

Step-by-step explanation:

What is m RMS in the figure below?
RA
P
M
128°
T
A 26
B 38
C 52
D 128

Answers

Answer:

Option (B)

Step-by-step explanation:

There are two lines PS and QT intersecting each other at a point M.

It's given that m∠PMT = 128° and m∠QMR = 90°

Since ∠PMT and ∠QMS are the vertical angles so these angles will be equal in measure.

∠PMT = ∠QMS

128° = m∠QMR + m∠RMS

128° = 90° + m∠RMS

m∠RMS = 128° - 90°

m∠RMS = 38°

Therefore, Option (B) will be the answer.

15+21 distributive property

Answers

The answer is 36

Explanation

Solve for x. Round if needed. Image is below.

Answers

Answer:

x = 16

Step-by-step explanation:

3x + 1 = 49 ( vertical angles )

Subtract 1 from both sides )

3x = 48 ( divide both sides by 3 )

x = 16

Is the number a rational number? Help me out please!

Answers

Answer:

first answer

Step-by-step explanation:

Hi, this is correct because repeating decimals are rational numbers.

Rational numbers can be written by a fraction of integers.

If there is a pattern, it means that this is the division of two integers.

hope this helps

Answer:

yes

Step-by-step explanation:

What is the justification for the step taken from line 2 to line 3?

Answers

Answer:

They subtracted 2x from both sides.

Step-by-step explanation:

They removed 2x from each side because in order to find x it will have to be removed on one side. Since 2x - 2x = 0 it is 2x is out of the equation for the right side and on the left side -2x -(-4x) = -6x which means that there is one x variable left in the equation thus making it solvable.

please someone help me to prove this..​

Answers

Answer:  see proof below

Step-by-step explanation:

Use the following Sum to Product Identities:

[tex]\sin x+\sin y=2\sin \bigg(\dfrac{x+y}{2}\bigg)\cos \bigg(\dfrac{x-y}{2}\bigg)\\\\\\\sin x-\sin y=2\cos \bigg(\dfrac{x+y}{2}\bigg)\sin \bigg(\dfrac{x-y}{2}\bigg)\\\\\\\cos x+\cos y=2\cos \bigg(\dfrac{x+y}{2}\bigg)\cos \bigg(\dfrac{x-y}{2}\bigg)\\\\\\\cos x+\cos y=-2\sin \bigg(\dfrac{x+y}{2}\bigg)\sin \bigg(\dfrac{x-y}{2}\bigg)[/tex]

Proof LHS → RHS

[tex]\text{LHS:}\qquad \qquad \qquad \dfrac{\sin 5-\sin 15+\sin 25 - \sin 35}{\cos 5-\cos 15- \cos 25 + \cos 35}[/tex]

[tex]\text{Reqroup:}\qquad \qquad \qquad \dfrac{(\sin 25+\sin 5)-(\sin 35 + \sin 15)}{(\cos 35+\cos 5)-(\cos 25 + \cos 15)}[/tex]

[tex]\text{Sum to Product:}\quad \dfrac{2\sin \bigg(\dfrac{25+5}{2}\bigg)\cos \bigg(\dfrac{25-5}{2}\bigg)-2\sin \bigg(\dfrac{35+15}{2}\bigg)\cos \bigg(\dfrac{35-15}{2}\bigg)}{2\cos \bigg(\dfrac{25+15}{2}\bigg)\cos \bigg(\dfrac{25-15}{2}\bigg)-2\cos \bigg(\dfrac{35+5}{2}\bigg)\cos \bigg(\dfrac{35-5}{2}\bigg)}[/tex][tex]\text{Simplify:}\qquad \qquad \dfrac{2\sin 15\cos 10-2\sin 25\cos 10}{2\cos 20\cos 15-2\cos 20\cos 5}[/tex]

[tex]\text{Factor:}\qquad \qquad \dfrac{2\cos 10(\sin 15-\sin 25)}{2\cos 20(\cos 15-\cos 5)}[/tex]

[tex]\text{Sum to Product:}\qquad \dfrac{\cos 10\bigg[2\cos \bigg(\dfrac{15+25}{2}\bigg)\sin \bigg(\dfrac{15-25}{2}\bigg)\bigg]}{\cos 20\bigg[-2\sin \bigg(\dfrac{15+5}{2}\bigg)\sin \bigg(\dfrac{15-5}{2}\bigg)\bigg]}[/tex]

[tex]\text{Simplify:}\qquad \qquad \dfrac{\cos 10[2\cos 20\sin (-5)]}{\cos 20[-2\sin 10\sin 5]}\\\\\\.\qquad \qquad \qquad =\dfrac{-2\cos10 \cos 20 \sin 5}{-2\sin 10 \cos 20 \sin 5}\\\\\\.\qquad \qquad \qquad =\dfrac{\cos 10}{\sin 10}\\\\\\.\qquad \qquad \qquad =\cot 10[/tex]

LHS = RHS:  cot 10 = cot 10   [tex]\checkmark[/tex]

Daryl is twice the difference of Amy’s age and 5. If their combined age is 50, how old is Darly

Answers

Answer:

30

Step-by-step explanation:

The following graph shows a proportionality between y and x in the graph ?

Answers

Answer:

3/1 is the ans

Step-by-step explanation:

here y is in 3

and x is in 1

so proportonality will be 3/1

Convert the decimal to a fraction. 0.185

Answers

Answer:

37/200

Step-by-step explanation:

.185= 185/1000

reduce to- 37/200

Hope this helps.

pls show work and explain how to do it

Answers

Step-by-step explanation:

1. We can see that the two vertical lines are parallel. Since one angle is 80º, then the other side should be 100º, since the straight line is 180º.

2. If the angle on the right is 80º, then that means the angle with y and 40º add up to 100º.

3. 100º - 40º = 60º.

Our final answer: y = 60º

I hope this helps!

Please help me with this!! (20 points)

Answers

Answer:

BC, BD

Step-by-step explanation

If right please brainly

Idk how to solve these

Answers

i think it’s 7, angle GHE

What would happen if I double the 4 and the 6 in 4-6

Answers

The answer is -4. The answer just is multiplied by 2.

Other Questions
A runner competed in a 5-mile run. How many yards did she run?a-8800 yardsb-8800 miles-8657 yards 8. Economists assume that everyone acts in their ownself-interest. Explain the difference between self-interest andselfishness. Helppp!! what are the correct abbreviation for Monsieur et Madame Dupont? In Zambia, which lions are likely to have migrated from a different population? If the velocity of a proton is straight up (thumb pointing up) then RHR2 shows that the force points to the left. What would the direction of the force be if the velocity were Solve the equation below for y.8x+4y=20 What are some new challenges today you have encountered as a college student? Midpoint of the segment with endpoints (-8, 5) and (3, -6) You have $100,000 to donate to your college. You want to endow a perpetual scholarship that makes its first payment in 1 year. If the college's discount rate is 5%, how large will the annual scholarship payment be? Bonita Company in its first year of operations provides the following information related to one of its available-for-sale debt securities at December 31, 2020.Amortized cost $50,900Fair value 41,800Expected credit losses 12,500Required:a. What is the amount of the credit loss that Bonita should report on this available-for-sale security at December 31, 2020?b. Prepare the journal entry to record the credit loss, if any (and any other adjustment needed), at December 31, 2020. c. Assume that the fair value of the available-for-sale security is $54,800 at December 31, 2020, instead of $41,800. What is the amount of the credit loss that Bonita should report at December 31, 2020?d. Assume the same information as for part (c). Prepare the journal entry to record the credit loss, if necessary (and any other adjustment needed), at December 31, 2020. What number comes next 57, 49, 41, 33, 25_ Shiro enrolled in Composition 101, a three-credit course. If this class meets twice a week, how long should Shiro expect each class to last Which is the lean principle where managers and employees seek to uncover waste in business activities including accounting activities such as payroll and disbursements? HELP ME ON THIS QUESTION PLEASE HELP ME I WOULD REALLY APPRECIATE IT THANK YOU Students are planning the backdrop for the school play. The backdrop is 15 feet wide and 10 feet high. Every 16 inches on the scale drawing represents 5 feet on the backdrop What is the area of the scale drawing? How do I solve an absolute value question? What is the correct answer with the appropriate significant digits?6.25 x 19.50 What is 2x=4???????????? 8. Joe walked 5 miles north, 9 miles east, then 9 miles north, and 7 miles east. If Joe decides towalk straight back to where he started, how far must he walk? From which country did the first settlers come to the United States?FranceEnglandO PortugalSpain